« first day (839 days earlier)      last day (4186 days later) » 

12:00 AM
It has interesting residues to figure out! :)
 
I can't see how though
 
@jdoe Wait, maybe not - I notice that the lower bound is 1
 
could it be holomorphic on the whole complex plane with lower bound 1?
$ = \int_1^\infty e^{(x-1)(\log t) - t} dt$
I suppose we could argue that whatever complex constant x is, -t dominates (x-1)(log t) so basically it's e^{-t} for large t
 
$$\frac{e^{(z-1)\log t}}{e^t}$$
 
12:10 AM
Your integral with the lower bound of $1$ can be written as $\Gamma(x, 1)$ if that is helpful (Incomplete Gamma Function).
 
well I think we can prove it directly because if t = e^y then that fraction (the integrand) is <= 1 whenever e^y/y > |z-1|
oh wait that's not enough because the limit is infinite
I mean the top is "polynomial" and the bottom is exponential so it must converge
 
No poles in the integrand...
 
oh yeah the integrand is holomorphic so its primitive will be
 
Also, while this is not rigourous, it is notable that for every $t$ between $1$ and $\infty$, the function is never indeterminate like in the regular gamma function w/ a lower bound of $0$.
 
what you mean like intederminate?
 
12:15 AM
i.e. $\frac{e^0}{0}$ in the integrand when $\Gamma(0)$
 
oh
so the integral definition of gamma works for all complex z with Re z > 1?
 
I believe so. 1 sec
@WillHunting Hi again. How was breakfast?
 
user19161
@Argon OK. I might go out to the park later for a walk. I have not been to that one for over a year...
 
@jdoe $\Re z > 0$ if memory serves...
 
@WillHunting Go ahead!
 
12:17 AM
@J.M., ah, that's good
@J.M., trying to continue it with this sum + integral is actually veryhard
because I cant show the integral converges
it obviously does but I don't know how ot proveit
 
user19161
@jdoe What did you use to plot that?
 
@WillHunting, I got it from wikipedia
> English: A 3D plot of the absolute value of the Γ function, self-made in Mathematica (prior versions in MuPAD)
 
user19161
@jdoe HAHAHAHAHA
 
what
 
user19161
12:21 AM
Nothing, I just did not expect it. =)
 
Let s be any complex number show that $$\int_1^\infty e^{-t} t^s dt$$ converges
 
Hmmm... I need to get smarter at this.
 
ah i got it
let s = x + iy
 
Ok
 
user19161
@j.m. I am wondering if you have any math books that are out of print today.
 
12:24 AM
$|t^s| = |t^x||\exp(i y \log(t))| = |t|^x$
 
user19161
@amwhy I got the Strunk and White badge now! That's for editing 80 posts.
 
so the integrand $\le |e^{-t}||t|^{\Re{s}}$
 
@jdoe $\log z= \operatorname{Log} |z|+i\theta$
@WillHunting Congratz
 
I showed the integrand is bounded by $x e^{x \log(x)} e^{\log u - u}$ with substitution $xu = t$
So just need to show that $$\int_1^\infty e^{\log u - u} du$$ converges
 
@WillHunting Yes, mostly yellowed...
One of these days, I should scan them before they become dust...
 
12:31 AM
could split that into $\frac{u}{e^{u/2}}\frac{1}{e^{u/2}}$ where the first factor is < 1 for large enough u and the second factor decreases exponentially which we need for the integral to converge
 
user19161
@J.M. Could you name one famous one?
 
I think Jahnke-Emde is pretty famous...
 
wait a sec how do we know the gamma integral converges for anything..
Let $$\Gamma(s) = \int_0^\infty e^{-t} t^{s-1} dt$$ this integral converges for $\Re (s) > 0$.
 
@WillHunting Yes, I've got that badge. I think I may even have a gold badge for editing 500 some posts awhile back!
 
@jdoe $\Gamma(1)$ converges trivially.
We show the rest of the integers converge by repeated integration by parts
 
user19161
12:36 AM
@amWhy Yes, I just saw that you did over 700!
 
@jdoe $\Gamma(1)$ converges trivially.
 
@Argon, but what about between integers and complex values?
 
We show the rest of the integers converge by repeated integration by parts
 
@WillHunting Wow, didn't realize it was that high, and I was gone for nearly a year (from math.se).
 
user19161
@amWhy You can check this out on the users page, just click on the editors tab and sort by all time.
 
12:37 AM
@WillHunting I learned LaTeX by editing
5
 
user19161
@amWhy Haha. Do you install TeX yourself on your own machine?
 
@jdoe The paper I linked you to yesterday had a proof
@amWhy Me too :)
 
@WillHunting I've got it installed, but can't figure out how to work with it. I've got to get back on that task!
 
@Argon, it says:
> Clearly, the second integral converges for any complex z
lol :D
 
@jdoe Hahaha
 
user19161
12:39 AM
@amWhy Haha, OK you can ask me if you run into problems. I recommend TeX Live over MikTeX.
 
at least it's clear to someone
 
@jdoe "It can be shown..."
@WillHunting LyX works for me but MikTeX never does - though it is needed by LyX!
 
@WillHunting Thanks! I'll look into TeX Live. I've got MikTeX installed, but haven't really tackled it yet. I need to learn how to "begin" a document (headers, etc.). I know the math part, it's the rest that I need to learn.
 
user19161
@Argon Oh, LyX is an editor and MikTeX is a TeX distribution, so they are different things. I use TeXworks and TeX Live.
 
@WillHunting LyX forced me to install it
 
user19161
12:42 AM
@Argon To install MikTeX?
 
@WillHunting Yep
 
how do i show there's a t' such that for all t >= t', $k \log t < t$
 
user19161
@Argon Anyway, I don't like LyX like I told you.
 
@WillHunting I know. But I do!
 
user19161
@Argon Good for you. I use TeXworks like the great anon!
 
12:43 AM
@WillHunting The great anon!
 
user19161
@Argon TeXworks: it just works!
 
user19161
HAHAHAHAHA
 
@WillHunting You sound like you work for them :)
 
user19161
@Argon I did try LyX once, it was just too weird, neither here nor there but in between.
 
@WillHunting So? It's an editor that allows TeX to be inserted as necessary!
 
user19161
12:45 AM
@Argon It's more like between a word processor and an editor.
 
@WillHunting And awesome
 
user19161
@Argon I also love the user interface of TeXworks, code on the left half and output on the right, and the ability to jump to and fro.
 
@WillHunting Never actually used it, so I cannot comment
 
user19161
@Argon The great Jonas uses Emacs.
 
@WillHunting Yay
@WillHunting What is it?
 
12:47 AM
Hi Amy!
 
user19161
@Argon An editor that is even harder than TeXworks.
 
user19161
@J.M. Hehe, at first I was wondering who you were referring to, then I learnt about it a few days ago...
 
@WillHunting When you are that good, I guess you can use it!
 
user19161
@Argon When you told me your last name, I understood why you are called Argon. =)
 
@WillHunting Marilia made the same realization :)
 
user19161
12:57 AM
@Argon I should be called Joy then. =)
 
@WillHunting Hahahaha Ya
 
I'd like to study the pochhammer contour continuationof beta
 
user19161
By the way, what happened to Mr Mick?
 
@jdoe ...I would too... If I could understand it!
That's a problem with math - I don't understand it :)
@WillHunting A very good question!
 
@Argon, yes wwe can both understand it should be easy
$$
\displaystyle (1-e^{2\pi i\alpha})(1-e^{2\pi i\beta})B(\alpha,\beta) =\int_C t^{\alpha-1}(1-t)^{\beta-1} \, dt. $$
C is anything like that
bye
 
1:04 AM
Bye
 
1:18 AM
@J.M.: Hey there... I've been away from chat for most of the day.
Are things slow here for others, or is it just local?
 
Local
 
@Link drat. I end up waiting for stackexchange.com and mathjax.org, so I thought it might be something on the other end.
 
@robjohn Do you have any tricks for deciding on what contour to use in integration?
 
@robjohn Hey. Nothing unusual, it looks.
 
@Argon It is a matter of looking at singularities, branch cuts and where the function is well behaved.
@Argon and sometimes, you can use different contours to get the same integral.
 
1:23 AM
@robjohn I always end up choosing a bad contour and then I don't know if it's my fault or the function on the contour :)
 
(i.e., don't have your contours cross regions where the function is wonky...)
 
@J.M. important safety tip :-)
 
@J.M. I learned that the hard with with hyperbolic functions, hahaha!
They act like circular trig functions on the complex axis!
So I often use a big rectangle for those
 
@Argon That's actually typical for a number of special functions: oscillatory in one region, exponentially growing/decaying in another...
(More often than not, you want the regions where it's decaying)
 
I have trouble w/ branch cuts and points as well...
How do you know where the cut is? e.g. with $\log$, must the cut run along the real axis?
 
1:28 AM
@Argon The principal branch of the logarithm is cut on the negative real axis.
 
@J.M. Why is that? Could I integrate it so I avoid, say, the positive complex axis instead?
 
@Argon It's more a convenience than anything. Of course, there's nothing preventing you from spinning the branch cut around so it ends up along a different ray...
 
@J.M. So should I think of it as a spiral, like the Riemann Surface, where I just need to make the function 1 to 1 by cutting somewhere?
 
Yes, the Riemann surface is an excellent way of looking at it. Since the logarithm is multibranched, you need to slice a particular section to get the one-to-one feature.
 
Neat. Because $a^b = \exp (b\log a)$, it follows that some exponential functions are multivalued as well. Which ones are? Why don't I take branch cuts for $e^x$?
 
1:33 AM
So, you'll want something like $\log |z|+i(\arg(z\exp(i\theta))-\theta)$ if you need the cut to be somewhere else...
@Argon The exponential function is entire/analytic/holomorphic, so no cuts...
@Argon Yes, power functions for noninteger powers should also inherit the logarithm's branch cut.
 
@J.M. So is $\exp$ the exception?
 
@Argon I'm making a distinction between exponential functions and power functions, to be clear. For $a^b$, $a$ is the one varying in a power function. For an exponential function, it's $b$ who's varying.
 
@J.M. I see, thanks. So $f(x)=x^a$ has cuts?
 
@Argon For noninteger $a$, yes.
 
@J.M. Why not integers?
 
user19161
1:39 AM
@J.M. Wow, this is the first time in my life I see "noninteger" I think! =)
 
@Argon If they weren't, polynomials wouldn't be entire...
A constant function $z^0$ is obviously entire; $z$ can be shown to be entire; and the product of two entire functions is also entire, so...
 
user19161
Interestingly, Lang uses the term "entire ring".
 
@J.M. That is very strange! :)
 
For, say, $1/z$, it's meromorphic (pole at zero), but no cuts...
 
But $z^1 = \exp(\log |z|+i\theta)$! Why does the argument not make multiple values?
I can say I'm still not entirely comfortable manipulating complex numbers like I am with reals...
 
user19161
1:43 AM
@argon Check your mail.
 
@WillHunting Recieved, sir!
 
user19161
@Argon receive
 
@WillHunting I wish I understood something there :'(
 
user19161
@Argon It is trivial, just thought I would send you for fun. You can read it in future. =)))
 
user19161
@Argon The only complex analysis I remember now is the definition of a complex number. =)
 
1:46 AM
@WillHunting $z=x+iy=Re^{i\theta}$. QED
 
user19161
@Argon Hahaha.
 
@Argon ...and what's $|z|\exp(i\arg\,z)$ supposed to be? :)
(remember the exponential function's periodicity as well)
 
user19161
@argon I can't view what you sent me as I don't have an account, you can send me the PDF if you want.
 
On that note: you should be able to recall one rather inconvenient feature of the polar coordinate system...
 
user19161
@J.M. Which is?
 
1:50 AM
@WillHunting To illustrate: why are $(1,\pi/4)$ and $(1,-7\pi/4)$ the same point?
 
@J.M. Hmmm... what even is $\exp(i\arg z)$? $z$ where $|z|=1$? (Don't laugh too hard)
 
user19161
@J.M. OIC. Yes I can still understand that. =)
 
Okay, answer my question on polar coordinates first. :) (Seems Will got it.)
 
Principle valueness
Periodicity
 
$\exp$ is $2\pi i$-periodic, yes?
 
1:52 AM
Yep
@WillHunting Check your electronic inbox!
 
So, whatever branch of the phase you pick, it shouldn't matter within the exponential.
Thus, you always have the polar decomposition $z=|z|\exp(i\arg\,z)$
 
So let us now consider $\sqrt{z} = \exp\left(\frac{1}{2}(\log |z|+i\arg z)\right)$
 
user19161
@Argon Received. Note the spelling again!
 
@WillHunting Spelling??
$\sqrt{|z|}\exp(i\arg z/2)$?
 
@Argon You lose the periodicity now; think of what happens after adding or subtracting an integer multiple of $2\pi$ from the argument...
 
user19161
1:55 AM
@Argon The spelling of "receive" which is not "recieve".
 
@WillHunting Meh
@J.M. $\exp(i\arg z/2+i\pi)$?
 
@Argon That'd correspond to the negative square root, no?
 
@J.M. Yep :)
Cool! $\exp(i\pi)=-1$
Thanks!
 
(It should now be clear why we have to pick which of the positive or negative square root should be principal for our purposes...)
 
Cool, thanks :)
My favourite show, Midsomer Murders, is on now. Good night!
 
2:00 AM
Anyway, I must go. See y'all later.
 
@J.M. Bye sir!
 
user19161
@Argon Oh noes, now you call everyone sir like iyengar!
 
@WillHunting Iyengar?
 
user19161
@Argon OK go watch your TV.
 
user19161
@Argon A user who came to chat long ago.
 
2:01 AM
@WillHunting Hahaha :)
@WillHunting He said sir then, I presume
 
user19161
@Argon The logic is obvious. QED.
 
@WillHunting Enjoy Weierstrass substitution. $\blacksquare$
Good night
 
user19161
@Argon See you in your dreams.
 
leo
2:17 AM
Are manifolds normal topological spaces?
 
user19161
@leo Yes, of course.
 
user19161
The usual definition is that it is a topological space that is Hausdorff and locally homeomorphic to Euclidean space.
 
Hello again
 
user19161
@leo Oh, sorry I misread the normal there. I thought you meant ordinary.
 
@leo: do you require manifolds to be second countable?
 
user19161
2:27 AM
@wj32 Ah, I missed that out too. =)
 
user19161
By the way @wj32 what does your username mean?
 
nothing
why does it have to mean something
 
 
1 hour later…
3:30 AM
0
Q: What's the meaning of the exponent here?

Gustavo BandeiraI'm reading A First Course in Logic: An Introduction to Model Theory, Proof Theory, Computability, and Complexity. The graph of $f: A \rightarrow B$ is the subset of $A × B$ consisting of all ordered pairs $(a, b)$ with $f (a) = b$. If $A$ happens to be $B^n$ for some $n ∈ N$, then we say...

 
user19161
4:16 AM
@GustavoBandeira Ah I wanted to answer that one but it's taken up already. =)
 
if I know the last M digits of fibonacci(N) is there a way I can extract Fibonacci(N)%b from this for arbitrary b?
 
4:41 AM
@WillHunting: you've studied algebraic topology, right?
 
 
3 hours later…
7:28 AM
@robjohn hi!
 
@N3buchadnezzar Hey there!
 
does anyone here know how to find the upper and lower bounds of a polynomial? (precalculus math)
 
what's up?
@MatthewWong for an odd order polynomial, there won't be either
 
@robjohn I have an integral, that I am supposed to optimize using linear algebra. Got time give it a glimpse? My book does not cover it
 
@N3buchadnezzar what integral?
 
7:30 AM
@robjohn by odd do you mean number of coefficients or the power?
 
@MatthewWong the order of a polynomial is the degree of the highest term: $x^3+x$ is third degree
 
?! if there isn't either then why does the textbook list it as a question?
 
@MatthewWong The upper and lower bounds for the real roots of the polynomials
 
oh i guess i should've mentioned that too
my bad :P
 
@MatthewWong what does Example 2 say?
 
7:35 AM
$$ \int_0^1 \left| t^4 - a - bt \right|^2 \,\mathrm{d}x$$
Find $a$ and $b$ to minimize.
 
i wish i knew or else i wouldn't have to ask. (i tried doing a google search of the textbook and the page wasn't listed in the preview)
this is a photocopy made by the teacher
 
According to the solution to a few similar problems they use it as a inner product, to calculate the projection from the space a + bt onto the function t^4. But I a having problems figuring out the details, as the integral does not define any inner product?
 
@N3buchadnezzar can you compute $\frac{\partial}{\partial a}$ and $\frac{\partial}{\partial b}$ of that integral?
@MatthewWong what is the text book?
 
@robjohn Precalculus: With Unit Circle Trigonometry
 
@robjohn No?
 
7:45 AM
$\frac{\partial}{\partial a}\int_0^1(t^4-a-bt)^2\,\mathrm{d}t=-2\int_0^1(t^4-a-bt)\,\mathrm{d}t$ and $\frac{\partial}{\partial b}\int_0^1(t^4-a-bt)^2\,\mathrm{d}t=-2\int_0^1(t^4-a-bt)t\,\mathrm{d}t$
set both to $0$ and solve for $a$ and $b$
 
I thought one needed the abs value
 
@N3buchadnezzar what is $|x|^2$?
 
@robjohn nevermind i found a powerpoint that helps me
 
@MatthewWong yeah, without knowing what Example 2 says, it is hard to know what to do.
 
@robjohn =) Gives me $a=-4/5$, $b = 1/5$ and $I_{a,b} = 4/225$
I guess I have to ask my teacher about the linear algebra approach to this problem
 
7:50 AM
fuck me...i have to list all the factors of 100 and synthetically divide them...
 
@N3buchadnezzar well, it is the inner product of $t^4-a-bt$ with itself
@MatthewWong to bound the real roots?
@MatthewWong that sounds like what you'd do to find the rational roots
 
yeah except it doesn't matter if theres a remainder or not
just quoting here
"Divide f (x) by x - b (where b > 0) using synthetic division. If the last row containing the quotient and remainder has no negative numbers, then b is an upper bound for the real roots of f (x) = 0."
"Divide f (x) by x - a (where a < 0) using synthetic division. If the last row containing the quotient and remainder has numbers that alternate in sign (zero entries count as positive or negative), then a is a lower bound for the real roots of f (x) = 0.
"
sounds a bit too easy tough o_o
*though
 
8:43 AM
Good day!
Hi @robjohn! Did you hear about nonnegative distributions (generalized functions)?
 
@Nimza what about them? did something happen in the news?
 
@robjohn no, I can't find anything on them. But I was told that there's some Bochner characterisation theorem for them...
 
@Nimza where are you seeing them?
 
@robjohn yesterday on seminar orator was speaking about them, but he thought that such basic things are classical and ommited them
 
@Nimza I don't know if this has anything to do with it.
 
8:56 AM
@robjohn yeah, looks like very similar to it! And I've found something in Vladimirov's book "Generalized functions in mathematical physics"
 
@Nimza Those distributions are a bit different, but I guess in certain usages, they could be used in a similar way
 
@robjohn aha, thank you for help :)
 
9:26 AM
Hmm
What does $\tilde{x}(t)$ mean ?
 
10:01 AM
@N3buchadnezzar It depends on the context
 
10:54 AM
Kay
 
@N3buchadnezzar what is the context that you have?
 
IT is in regard to functional analysis, and metric spaces.
Problem 3
 
11:25 AM
@N3buchadnezzar In that question, it just seems to denote a second value. That is $x$ and $\bar{x}$ are like $x_1$ and $x_2$ or $x$ and $x'$.
@N3buchadnezzar admittedly, that was not one of the uses I've seen for $\bar{x}$
Wow, we are down to 4...
 
 
1 hour later…
12:36 PM
is there a picture of Michael Spivak out there?
 
1:21 PM
Hi, I'm currently a student in an online AP BC calc class, and I'm having a little trouble with a question about derivatives and parametric equations - I emailed by teacher, but I think she's on an unofficial early thanksgiving break -.- (We are assigned enough work regardless to have to work until friday, but she obviously won't be there - and all assignments are due then)

Anyways, the problem is written as follows:
"The position of an object is described by the parametric equations x=ln t and y=5t^2. What is the acceleration of the object in /sec^2 when t=2?
I've been wondering the whole time what I should be calculating - my sister says I should be calculating d^2y/dx^2, but personally I think I should be calculating sqrt(d^2y/dt^2+d^2x/dt^2) for t=2 - I'm a little confused, because I thought acceleration had direction as well, meaning that a single scalar would not accurately describe the acceleration of a particle with motion described in x and y axis?

Thanks guys.
 
1:36 PM
hello
 
> The position of an object is described by the parametric equations x=ln t and y=5t^2. What is the acceleration of the object in /sec^2 when t=2
hmm
is acceleration a vector (with x,y components) or a magnitude?
> because I thought acceleration had direction as well, meaning that a single scalar would not accurately describe the acceleration
I agree with you here
let's say acceleration is a vector with x and y components
@user1230219, the position of the object at time t is (ln t, 5t^2) so it's velocity is ( d/dt ln t, d/dt 5t^2 )
 
And acceleration is the second derivative correct?
The question is, is my teacher really looking for this, or simply looking for d^2y/dx^2?
I mean, the magnitude of acceleration.
calculating the second derivative for the x and y components, I get
d^2x/dt^2=e^x
d^y/dy^2=10
 
do you agree velocity is ( d/dt ln t, d/dt 5t^2 )
 
To be quite honest, I'm not sure - intuitively yes, I guess. Are we missing a direction component here?
 
1:49 PM
ok so lets start at basics
We have an object whose position at time t is given by (x(t),y(t)) and those are the x,y coordinates
so in this case the object is at (ln 2, 5*4) when t=2
 
Okay.
 
and at t=3 it's at (ln 3, 5*9)
so in one second it has moved this much: (ln 3 - ln 2, 5*(9-4))
so for t between 2 and 3, it's velocity is about 0.4 m/s in the x axis and 25 m/s in the y-axis
 
I see
 
ok good!
so do you know why I just said velocity is 'about'
rather than exactly
 
It's not instantaneous?
 
1:56 PM
yeah
 
$f(t) \cdot \overline{f(t)} = ?$
 
complex conjugate?
 
@user1230219, so what would give the exact velocity at some time t?
 
@peoplepower Indeed, is there any easier way to write it ?
$$|f(t)|^2$$
 
@N3buchadnezzar Is it not $|f(t)|^2$?
 
1:58 PM
Silly me
 
Silly me too, I forgot to square it.
 
@jdoe, sorry for my delayed reponses - I made a post in math.stack exchange and am reading feedback
the exact velocity at some time t would be (x'(t),y'(x)), correct?
 
@user1230219, no y is a function of t
 
Er, typo there
 
@peoplepower Thanks anyway =)
 
2:05 PM
Correction:(x'(t),y'(t))

and the acceleration would be (x''(t),y''(t))?
 
@user1230219, yeah differentiating with respect to time, it's velocity being how the position (in each coordinate) changes with time
@user1230219, so you can compute it ( d/dt ln t, d/dt 5t^2 ) = (1/t, 10t)
 
The only problem is that I am requested to give units in the form m/sec^2 - again, I'm guessing that they want magitude correct?
http://math.stackexchange.com/questions/241312/acceleration-of-a-particle-described-by-parametric-equations
 
so what is the acceleration
 
The second derivative?
 
yes but compute it
 
2:09 PM
well, I have two parametric equations
x=ln t and y=5t^2
d^2x/dt^2=-1/(t^2)
d^2y/dt^2=10
 
great so acceleration is?
 
so i evaluate for t=t, the eppression sqrt(10^1+(-1/4)^2)?
 
no
 
Well, the problem is phrased as wanting units in m/sec^2...
 
position is (ln t, 5t^2), velocity is (1/t, 10t)
 
2:12 PM
Hey, I'm really sorry - the bell just rang, so I have to get to my next class
thanks for helping :)
 
2:44 PM
darn I forgot t look up the pochhammer thing at the library today
hi
 
And then Thor used the POCHHAMMER! And peace once lay over Valhalla.
 
3:30 PM
Artin defines rings with unity right?
 
@N3buchadnezzar and loki is now spreading the word?
 
@JayeshBadwaik y
 
if f is in L^1 then is sup_[a,b] |f(t)| defined?
 
sup |f| is the L^1 norm right?
 
I don't even know :(
 
3:44 PM
yes I believe so
wait, that's the L^infty norm
the L^1 norm is int |f| dt
 
so ill have to say f is in L^1 and bounded
but how would I show its fourier transform $$\hat f (\lambda) = \int_{-\infty}^\infty f(t) \exp(- i \lambda t) dt $$ is bounded
oh that's easy just use |integral fg| <= integral |f||g|
 
user19161
4:08 PM
@anon Yes.
 
user19161
@N3buchadnezzar Haha, Andrew is your lecturer?
 
@WillHunting Hi. Why did you ditch Ubuntu? (just asking, because I just upgraded my dad's computer to Ubuntu 12.04) Its pretty good. I even donated some money to Canonical as a fee.
 
Can you help me show $\hat {\hat f(t)} = 2 \pi f(-t)$
using $$E(x) = e^{-\frac{x^2}{2}}$$
I think it just comes from $\hat E = \sqrt{2 \pi} E$ and $\int \hat f g = \int f \hat g$
 
Parsevals
 
4:24 PM
I need $$\hat {\hat f(t)} = 2 \pi f(-t)$$ to prove parseval
 
one way is to use the fact about orthogonality of $exp(-i\lambda_1 t)$ and $exp(-i \lambda_2 t)$ , $\lambda_1 \neq \lambda_2$ and then some computations.
 
@WillHunting He is on vacation, but if he werent he would be.
 
so far I got $$\int \hat f(t) E(t/R) = R \sqrt{2 \pi} \int f(\lambda) \hat E(\lambda/R)$$
as R tends to infinity LHS tends to integral of f hat, but I don't know what the RHS does
oops shouldn't have a hat on the RHS removing it is what gave me the 2pi
should I replace f with something orthogonal times f?
this jst gets more wrong the more I look at it
 
user19161
4:46 PM
@JayeshBadwaik (1) The default desktop, Unity, is unresponsive and ugly, even though one can install GNOME on top. (2) The only themes available in a default install are ugly, and others don't work well with Unity. (3) It cannot display the graphics driver correctly when other distros can. (4) All its innovations, like overlay scrollbars and HUD, are not to my liking. (5) I like independent distros more than dependent ones.
 
@WillHunting Hmm. Well, my dad uses KDE, so :D
 
user19161
@anon So does Bourbaki, Cohn, Rowen, Lang, Jacobson, MacLane. But not Herstein, Fraleigh, Grillet, Knapp, Malik.
 
user19161
@JayeshBadwaik Then just install Kubuntu instead of Ubuntu.
 
@WillHunting Its the same thing, except for installers, also, I had already installed the stuff, just upgraded in situ. I dislike clean installs.
@jdoe hmm, you are tying yourself up in tangles, write the rhs as a fourier transform and then again some similar stuff and you should be done.
 
user19161
@JayeshBadwaik Really? I like clean installs.
 
4:51 PM
i can't do it
 
@WillHunting Dude, the first rule they taught me at RHEL training course (which I attended for fun) was that you should avoid re-installs as far as possible. (Not giving any authoritarian credence to it. But this practise is actually true, you don't lose settings, you don't lose stuff, you basically hit the ground running.
 
why avoid reinstalls
 
user19161
@JayeshBadwaik The first thing I want to say about that is, it's all a matter of preference.
 
@jdoe check my updated answer.
 
where
 
4:54 PM
1 min ago, by Jayesh Badwaik
@WillHunting Dude, the first rule they taught me at RHEL training course (which I attended for fun) was that you should avoid re-installs as far as possible. (Not giving any authoritarian credence to it. But this practise is actually true, you don't lose settings, you don't lose stuff, you basically hit the ground running.
 
user19161
@JayeshBadwaik Upgrade if you don't want to reconfigure your system. Reinstall if you want to reconfigure your system. QED.
 
@WillHunting And you should not have to reconfigure your system too often.
 
user19161
@JayeshBadwaik Practice is the noun and practise is the verb in BrE.
 
@WillHunting hmm.... too late to modify now. Next time. Thanks.
 
user19161
@JayeshBadwaik I like to reconfigure once every two years when a new Debian release is out. Upgrades can go wrong, and you may need to download even more stuff and it may take even longer than a fresh install.
 
user19161
4:57 PM
If you are in the middle of an upgrade and the internet connectivity is not good, you are done for.
 
@WillHunting Depends on the system. Are you aware of the / -> /usr shift?
 
user19161
Maybe ten hours later when you are done downloading, the upgrade fails and you cry.
 
user19161
@JayeshBadwaik I know Fedora is changing its filesystem.
 
@WillHunting Well, thing is in ArchLinux, I was able to do it without reinstallation (without anything special actually), which shows how good upgrades can be when done well.
Also, I never had problems upgrading Debian or Ubuntu.
I have upgraded seamlessly from 8.04 to 10.04 to 12.04
 

« first day (839 days earlier)      last day (4186 days later) »